物理のかぎしっぽ 記事ソース/場のラグランジアンの使い方

記事ソース/場のラグランジアンの使い方

これはrst2hooktailの記事ソース保存・変換用です(詳細).

コンバート

最近コンバートされた結果: HTMLPDFTeX

公開・更新メニュー ▼▲

記事ソースの内容

============================================================
場のラグランジアンの使い方
============================================================

この記事では、中性スカラー場のラグランジアンから、
オイラー・ラグランジュ方程式を使って、運動方程式を導きます。
その計算過程を詳しく示すのが目的です。

ラグランジアンとオイラー・ラグランジュ方程式
================================================

ここではラグランジアン $\mathcal{L}$ を次の様に定めます。

<tex>
\mathcal{L} = \dfrac{1}{2}\partial_{\mu} \phi \partial^{\mu} \phi - \dfrac{1}{2} \mu^2 \phi^2 - \dfrac{1}{4!} \lambda \phi^4 \tag{##}
</tex>

これをオイラー・ラグランジュ方程式に代入します。それは次で示します。

<tex>
\dfrac{\delta S}{\delta \phi} = \dfrac{\partial \mathcal{L}}{\partial \phi} - \partial_{\nu} \left( \dfrac{\partial \mathcal{L}}{\partial (\partial_{\nu} \phi) } \right) = 0 \tag{##}
</tex>

計算の実行
=====================

さて、まず中辺第一項を求めます。これはまあそんなに難しくないです。

<tex>
\delta S_1 &= \dfrac{\partial \mathcal{L}}{\partial \phi} \delta \phi \\
&= \left(- \dfrac{1}{2} \mu^2 (\phi+\delta \phi)^2 - \dfrac{1}{4!} \lambda (\phi+\delta \phi)^4 \right) - \left( - \dfrac{1}{2} \mu^2 \phi^2 - \dfrac{1}{4!} \lambda \phi^4 \right) \\
&= - \left\{ \dfrac{1}{2} \mu^2 \left( (\phi^2+ 2 \phi \delta \phi) - \phi^2 + o(\delta \phi^2) \right) \right\} - \left\{ \dfrac{1}{4!} \lambda \left( (\phi^4+ 4 \phi^3 \delta \phi) - \phi^4 + o(\delta \phi^2) \right) \right\} \\
&= - \mu^2 \phi \delta \phi - \dfrac{1}{4!} 4 \lambda \phi^3 \delta \phi \\
&= - \mu^2 \phi \delta \phi - \dfrac{1}{3!} \lambda \phi^3 \delta \phi \tag{##}
</tex>

ですね。

だから、

<tex>
\dfrac{\delta S_1}{\delta \phi} &= - \mu^2 \phi - \dfrac{1}{3!} \lambda \phi^3  \tag{##}
</tex>

となります。ここでは微小量 $\delta \phi$ を用いましたが、これは要は $\dfrac{\partial}{\partial \phi}$ を作用させる事と同等です。次では $\delta (\partial_\mu \phi)$ を使うのが本来でしょうが、非常に見づらいので $\dfrac{\partial}{\partial (\partial \phi)}$ で計算することにします。さて、次が僕が戸惑ってしまった計算です。

<tex>
\dfrac{\delta S_2}{\delta \phi} &= - \partial_{\nu} \left( \dfrac{\partial \mathcal{L}}{\partial (\partial_{\nu} \phi) } \right) \\
&= - \partial_{\nu} \left( \dfrac{\partial}{\partial (\partial_{\nu} \phi)} \dfrac{1}{2} \partial_{\mu} \phi \partial^{\mu} \phi \right) \\
&= - \partial_{\nu} \left( \dfrac{\partial}{\partial (\partial_{\nu} \phi)} \dfrac{1}{2} g^{\mu \rho} \partial_{\mu} \phi \partial_{\rho} \phi \right) \\
&= - \partial_{\nu} \left( \dfrac{1}{2} g^{\mu \rho} \delta_{\mu}^{\nu} \partial_{\rho} \phi + \dfrac{1}{2} g^{\mu \rho} \partial_{\mu} \phi \delta_{\rho}^{\nu} \right) \\
&= - \dfrac{1}{2} g^{\mu \rho} \partial_{\mu} \partial_{\rho} \phi - \dfrac{1}{2} g^{\mu \rho} \partial_{\rho} \partial_{\mu} \phi  \\
&= - \dfrac{1}{2} \partial^{\rho} \partial_{\rho} \phi - \dfrac{1}{2} \partial^{\mu} \partial_{\mu} \phi  \\
&= -  \partial^{\mu} \partial_{\mu} \phi \\
&= - \Box \phi \tag{##}
</tex>

よって、

式 $(2)$ は、

<tex>
\dfrac{\delta (S_1+S_2)}{\delta \phi} &= - \mu^2 \phi - \dfrac{1}{3!} \lambda \phi^3 - \Box \phi = 0 \\
\left( \Box + \mu^2 \right) \phi &= - \dfrac{1}{3!} \lambda \phi^3 \tag{##}
</tex>

と書けるわけです。今日はここまで、お疲れさまでした!

@@reference: 九後汰一郎,ゲージ場の量子論I,培風館,1989,p21,4563024236@@

@@author:クロメル@@
@@accept:2019-03-28@@
@@category:量子力学@@
@@id:fieldDerivative@@
トップ   編集 凍結 差分 バックアップ 添付 複製 名前変更 リロード   新規 一覧 単語検索 最終更新   ヘルプ   最終更新のRSS
Modified by 物理のかぎプロジェクト PukiWiki 1.4.6 Copyright © 2001-2005 PukiWiki Developers Team. License is GPL.
Based on "PukiWiki" 1.3 by yu-ji Powered by PHP 5.3.29 HTML convert time to 0.008 sec.